25 votos

¿Asintótica errónea de la OEIS A000607 (número de particiones de un entero en partes primos)?

Secuencia A000607 en la Enciclopedia Online de Secuencias de Números Enteros es el número de particiones de $n$ en partes principales. Por ejemplo, hay $5$ particiones de $10$ en partes principales: $10 = 2 + 2 + 2 + 2 + 2 = 2 + 2 + 3 + 3 = 2 + 3 + 5 = 3 + 7 = 5 + 5.$ La OEIS da una expresión asintótica

$$A000607(n) \sim \exp\left(2 \pi \sqrt{\frac{n}{3 \log n}}\right). $$

Numéricamente, esto parece ser incorrecto incluso si se toma el logaritmo de ambos lados. Mi conjetura es que

$$\lim_{n \to \infty} \log\left(A000607(n)\right) \bigg/ \left( 2 \pi \sqrt{\frac{n}{3 \log n}} \right) \ne 1.$$

Véase el siguiente gráfico:

graph of logarithm of ratio

¿Cómo se puede probar o refutar esta conjetura?

Para más referencias, consulte http://oeis.org/A000607 .

0 votos

Hay un debate sobre las particiones en partes primarias en math.stackexchange.com/questions/89240/prime-partition (pero no se discute específicamente la asintótica).

3 votos

Más relevante es la discusión en mathoverflow.net/questions/69680/ donde se cita el libro de Flajolet y Sedgewick que tiene una prueba de (la forma logarítmica de) la asintótica citada de oeis.

42voto

Matt Puntos 8

Sus datos son compatibles con las estimaciones más refinadas demostradas por Vaughan en Ramanujan J. 15 (2008), 109-121. Sus teoremas 1 y 2 (junto con su (1.9)) revelan que $$\log(A000607(n)) = 2 \pi \sqrt{\frac{n}{3 \log n}}\left(1+\frac{\log\log n}{\log n}+O\left(\frac{1}{\log n}\right)\right). $$ Para $n=50000$ tenemos $$\log(A000607(n)) \approx 252.663 $$ $$ 2 \pi \sqrt{\frac{n}{3 \log n}} \approx 246.601$$ $$ 2 \pi \sqrt{\frac{n}{3 \log n}}\left(1+\frac{\log\log n}{\log n}\right)\approx 300.877$$ Por tanto, si se utiliza el término secundario que está presente en la fórmula de Vaughan, la aproximación (sin el término de error) no está por debajo sino por encima del valor real. También vemos que en este caso concreto el error es $\approx 48.214$ que es muy compatible con el hecho de que el término de error anterior es $O(1)$ veces $$ 2 \pi \sqrt{\frac{n}{3 \log n}}\cdot\frac{1}{\log n}\approx 22.792.$$

En resumen, su conjetura es probablemente falsa, mientras que Vaughan tiene razón. La anomalía numérica está causada por un término secundario que es bastante grande para el $n$ que has considerado.

1 votos

Muchas gracias, acepto que mi conjetura es falsa y el término asintótico menor es importante.

1 votos

No he revisado este documento cuidadosamente, así que no puedo asegurarlo, pero arxiv.org/pdf/1609.06497.pdf afirma que hay un error en el término de segundo orden de Vaughan, y que es $-\frac 12 \frac{\log \log n}{\log n}$ y la constante sobre $\log n$ término se identifica allí. Al menos, la respuesta del documento de física se ajusta mejor a los números.

0 votos

@Lucia: ¡Creo que los físicos tienen razón! Me parece que Vaughan ha cometido un simple error de cálculo en la segunda pantalla de la página 118. El coeficiente de $\log\log x$ debería haber $+1/2$ en lugar de $-1$ . Por lo tanto, todos los coeficientes de $\log\log x/\log x$ en las siguientes visualizaciones de esta página debe multiplicarse por $-1/2$ y esto corrige el Teorema 1 en armonía con el resultado de los físicos. ¿Puede confirmar esto? Si está de acuerdo, entonces actualizaría mi respuesta anterior y enviaría un mensaje a todos los autores (mencionándole a usted también).

i-Ciencias.com

I-Ciencias es una comunidad de estudiantes y amantes de la ciencia en la que puedes resolver tus problemas y dudas.
Puedes consultar las preguntas de otros usuarios, hacer tus propias preguntas o resolver las de los demás.

Powered by:

X